Add last few weeks of Phys 102 solutions.
[course.git] / latex / problems / Serway_and_Jewett_8 / problem29.40.tex
1 \begin{problem*}{29.40}
2 Consider the system pictured in Figure P29.40.  A $15.\U{cm}$
3 horizontal wire of mass $15.0\U{g}$ is placed between two thin,
4 vertical conductors, and a uniform magnetic field acts perpendicular
5 to the page.  The wire is free to move vertically without friction on
6 the two vertical conductors.  When a $5.00\U{A}$ curent is directed as
7 shown in the figure, the horizontal wire moves upward at a constant
8 velocity in the presence of gravity.  \Part{a} What forces act on the
9 horizontal wire, and \Part{b} under what condition is the wire able to
10 move upward at a constant velocity.  \Part{c} Find the magnitude and
11 direction of the minimum magnetic field required to move the wire at a
12 constant speed.  \Part{d} What happens if the magnetic field exceeds
13 this minimum value?
14 \end{problem*}
15
16 \begin{solution}
17 \end{solution}